LSAT and Law School Admissions Forum

Get expert LSAT preparation and law school admissions advice from PowerScore Test Preparation.

 angelsfan0055
  • Posts: 39
  • Joined: Feb 26, 2021
|
#95552
I'm wondering how B is the correct answer. I initially chose this answer but I thought it was an example of a shell game, because the conclusion focuses on businesses that want to remain power, not solely institutions. I was picking between B and E and since the conclusion solely focused on businesses remaining in power, I thought that B was a trap answer.

Could you explain how B is the correct even if it focuses on the broad institutions rather than business as articulated in the conclusion?
User avatar
 katehos
PowerScore Staff
  • PowerScore Staff
  • Posts: 184
  • Joined: Mar 31, 2022
|
#96049
Hi angelsfan0055!

The word institution here, as you mentioned, is broader than the term business, but I think you're overthinking this one! The word "institution" certainly applies to businesses, as they are institutions. You can even find evidence for this within the stimulus itself, as the Iron Law of Responsibility is said to specifically apply to "human institutions" by the author, who then discusses how this applies to businesses specifically. So, it still weakens the argument for the reasons addressed by the the other instructors.

I hope this helps! :)
Kate
User avatar
 lemonade42
  • Posts: 42
  • Joined: Feb 23, 2024
|
#105436
Hi Powerscore,

I know that I have to find an answer that allows the sufficient condition (WRP) to occur without the necessary condition (AR). So I chose (C) as the answer because I thought that "the power of some institutions erodes more slowly than the power of others, whether they are socially responsible or not" would mean this is possible: an institution whose power erodes more slowly (suggesting a business that wishes to retain its power as long as it can) that is not socially responsible. I thought that would be an example of the sufficient condition (WRP) occurring without the necessary condition (AR) and therefore weaken the argument.

Can you help explain why that's wrong? Thank you!
 Luke Haqq
PowerScore Staff
  • PowerScore Staff
  • Posts: 747
  • Joined: Apr 26, 2012
|
#105447
Hi lemonade42!

A problem with answer choice (C) is that it's not really giving us specific information about the institutions it's discussing. (C) is just saying the rate of erosion may be different for different companies, regardless of their social responsibility (e.g., perhaps bigger companies erode slower than smaller companies, or maybe vice versa). Even if that were the case, it could still hold true that to retain power for as long as possible, companies must act responsibly. (C) thus doesn't ultimately weaken this conclusion.

Get the most out of your LSAT Prep Plus subscription.

Analyze and track your performance with our Testing and Analytics Package.